Finding $limlimits_{n→∞}frac{ln(n)}nsumlimits_{k=1}^nfrac{a_k}k$












-2












$begingroup$


For a given sequence $a_1,a_2,a_3,…,a_n$, if $limlimits_{n→∞}=a$, find$$lim_{n→∞}frac{ln(n)}nsum_{k=1}^nfrac{a_k}k.$$
Can there be no question which a person is not even able to start? If I have no idea how to start a question how can I say what I did?










share|cite|improve this question











$endgroup$








  • 1




    $begingroup$
    It would be helpful if you worded your question a little more clearly. What is it you're trying to do?
    $endgroup$
    – Josh B.
    Dec 23 '18 at 2:36






  • 1




    $begingroup$
    You'll find that simple "Here's the statement of my question, solve it for me" posts will be poorly received. What is better is for you to add context (with an edit): What you understand about the problem, what you've tried so far, etc.; something both to show you are part of the learning experience and to help us guide you to the appropriate help. You can consult this link for further guidance.
    $endgroup$
    – Shaun
    Dec 23 '18 at 2:51
















-2












$begingroup$


For a given sequence $a_1,a_2,a_3,…,a_n$, if $limlimits_{n→∞}=a$, find$$lim_{n→∞}frac{ln(n)}nsum_{k=1}^nfrac{a_k}k.$$
Can there be no question which a person is not even able to start? If I have no idea how to start a question how can I say what I did?










share|cite|improve this question











$endgroup$








  • 1




    $begingroup$
    It would be helpful if you worded your question a little more clearly. What is it you're trying to do?
    $endgroup$
    – Josh B.
    Dec 23 '18 at 2:36






  • 1




    $begingroup$
    You'll find that simple "Here's the statement of my question, solve it for me" posts will be poorly received. What is better is for you to add context (with an edit): What you understand about the problem, what you've tried so far, etc.; something both to show you are part of the learning experience and to help us guide you to the appropriate help. You can consult this link for further guidance.
    $endgroup$
    – Shaun
    Dec 23 '18 at 2:51














-2












-2








-2


0



$begingroup$


For a given sequence $a_1,a_2,a_3,…,a_n$, if $limlimits_{n→∞}=a$, find$$lim_{n→∞}frac{ln(n)}nsum_{k=1}^nfrac{a_k}k.$$
Can there be no question which a person is not even able to start? If I have no idea how to start a question how can I say what I did?










share|cite|improve this question











$endgroup$




For a given sequence $a_1,a_2,a_3,…,a_n$, if $limlimits_{n→∞}=a$, find$$lim_{n→∞}frac{ln(n)}nsum_{k=1}^nfrac{a_k}k.$$
Can there be no question which a person is not even able to start? If I have no idea how to start a question how can I say what I did?







real-analysis






share|cite|improve this question















share|cite|improve this question













share|cite|improve this question




share|cite|improve this question








edited Dec 23 '18 at 3:54









Saad

19.7k92352




19.7k92352










asked Dec 23 '18 at 2:31









mavericmaveric

79112




79112








  • 1




    $begingroup$
    It would be helpful if you worded your question a little more clearly. What is it you're trying to do?
    $endgroup$
    – Josh B.
    Dec 23 '18 at 2:36






  • 1




    $begingroup$
    You'll find that simple "Here's the statement of my question, solve it for me" posts will be poorly received. What is better is for you to add context (with an edit): What you understand about the problem, what you've tried so far, etc.; something both to show you are part of the learning experience and to help us guide you to the appropriate help. You can consult this link for further guidance.
    $endgroup$
    – Shaun
    Dec 23 '18 at 2:51














  • 1




    $begingroup$
    It would be helpful if you worded your question a little more clearly. What is it you're trying to do?
    $endgroup$
    – Josh B.
    Dec 23 '18 at 2:36






  • 1




    $begingroup$
    You'll find that simple "Here's the statement of my question, solve it for me" posts will be poorly received. What is better is for you to add context (with an edit): What you understand about the problem, what you've tried so far, etc.; something both to show you are part of the learning experience and to help us guide you to the appropriate help. You can consult this link for further guidance.
    $endgroup$
    – Shaun
    Dec 23 '18 at 2:51








1




1




$begingroup$
It would be helpful if you worded your question a little more clearly. What is it you're trying to do?
$endgroup$
– Josh B.
Dec 23 '18 at 2:36




$begingroup$
It would be helpful if you worded your question a little more clearly. What is it you're trying to do?
$endgroup$
– Josh B.
Dec 23 '18 at 2:36




1




1




$begingroup$
You'll find that simple "Here's the statement of my question, solve it for me" posts will be poorly received. What is better is for you to add context (with an edit): What you understand about the problem, what you've tried so far, etc.; something both to show you are part of the learning experience and to help us guide you to the appropriate help. You can consult this link for further guidance.
$endgroup$
– Shaun
Dec 23 '18 at 2:51




$begingroup$
You'll find that simple "Here's the statement of my question, solve it for me" posts will be poorly received. What is better is for you to add context (with an edit): What you understand about the problem, what you've tried so far, etc.; something both to show you are part of the learning experience and to help us guide you to the appropriate help. You can consult this link for further guidance.
$endgroup$
– Shaun
Dec 23 '18 at 2:51










2 Answers
2






active

oldest

votes


















2












$begingroup$

Let's start with what we know. We are given that $lim_{n rightarrow infty}a_n=a$, but you will see that we don't actually need to use this. All that matters is that the sequence is bounded (convergence implies bounded). i.e.



$$exists ; M in Bbb R ; | ; |a_k|<M ; ; forall ; k in Bbb N$$



Also, the Euler constant should come in handy, and it is defined as $$gamma := lim_{n rightarrow infty}bigg(sum_{k=1}^nfrac 1k - ln(n)bigg)$$



and implies that



$$lim_{n rightarrow infty}bigg|sum_{k=1}^nfrac 1k - ln(n)-gammabigg|=0$$



Using L'Hopital's rule, we see that



$$lim_{n rightarrow infty}frac{big[ln(n)big]^2}{n}=0 qquad text{ and } qquad lim_{n rightarrow infty}frac{ln(n)}{n}=0 $$



Thus, we have:



begin{align}
& bigg|frac{ln(n)}{n}sum_{k=1}^nfrac {a_k}{k} bigg|\
leq & frac{ln(n)}{n}bigg|sum_{k=1}^nfrac {|a_k|}{k}bigg| \
leq & frac{ln(n)}{n}bigg|sum_{k=1}^nfrac {M}{k}bigg| \
= & Mfrac{ln(n)}{n}bigg(bigg|sum_{k=1}^nfrac {1}{k}-ln(n)-gamma bigg|+bigg|ln(n)+gammabigg|bigg) \
= & Mbigg(frac{ln(n)}{n}bigg)bigg|sum_{k=1}^nfrac {1}{k}-ln(n)-gamma bigg|+Mfrac{big[ln(n)big]^2}{n}+Mgamma frac{ln(n)}{n}\
rightarrow & M(0)|0| + M(0) + Mgamma (0) \
= & 0
end{align}



which implies that



$$lim_{n rightarrow infty}frac{ln(n)}{n}sum_{k=1}^nfrac {a_k}{k}=0$$






share|cite|improve this answer









$endgroup$













  • $begingroup$
    will there be a different method which does not involve so advance concepts
    $endgroup$
    – maveric
    Dec 23 '18 at 9:42










  • $begingroup$
    which of the above concepts do you consider advanced?
    $endgroup$
    – glowstonetrees
    Dec 23 '18 at 16:03



















1












$begingroup$

First consider the case where $a_k = a$ for all $k$. Then the sum becomes a sum of $1/k$, which can be approximated by comparing it to the integral of $1/x$ in some way that I forget but involves the Euler-Mascheroni constant.



Anyway I think you get something that converges to $0$, and then you can argue that if $a_k to a$ then it isn't different enough from the above case, so will also converge to $0$.






share|cite|improve this answer









$endgroup$













  • $begingroup$
    could not understand the last few lines.
    $endgroup$
    – maveric
    Dec 23 '18 at 3:04











Your Answer





StackExchange.ifUsing("editor", function () {
return StackExchange.using("mathjaxEditing", function () {
StackExchange.MarkdownEditor.creationCallbacks.add(function (editor, postfix) {
StackExchange.mathjaxEditing.prepareWmdForMathJax(editor, postfix, [["$", "$"], ["\\(","\\)"]]);
});
});
}, "mathjax-editing");

StackExchange.ready(function() {
var channelOptions = {
tags: "".split(" "),
id: "69"
};
initTagRenderer("".split(" "), "".split(" "), channelOptions);

StackExchange.using("externalEditor", function() {
// Have to fire editor after snippets, if snippets enabled
if (StackExchange.settings.snippets.snippetsEnabled) {
StackExchange.using("snippets", function() {
createEditor();
});
}
else {
createEditor();
}
});

function createEditor() {
StackExchange.prepareEditor({
heartbeatType: 'answer',
autoActivateHeartbeat: false,
convertImagesToLinks: true,
noModals: true,
showLowRepImageUploadWarning: true,
reputationToPostImages: 10,
bindNavPrevention: true,
postfix: "",
imageUploader: {
brandingHtml: "Powered by u003ca class="icon-imgur-white" href="https://imgur.com/"u003eu003c/au003e",
contentPolicyHtml: "User contributions licensed under u003ca href="https://creativecommons.org/licenses/by-sa/3.0/"u003ecc by-sa 3.0 with attribution requiredu003c/au003e u003ca href="https://stackoverflow.com/legal/content-policy"u003e(content policy)u003c/au003e",
allowUrls: true
},
noCode: true, onDemand: true,
discardSelector: ".discard-answer"
,immediatelyShowMarkdownHelp:true
});


}
});














draft saved

draft discarded


















StackExchange.ready(
function () {
StackExchange.openid.initPostLogin('.new-post-login', 'https%3a%2f%2fmath.stackexchange.com%2fquestions%2f3050018%2ffinding-lim-limits-n%25e2%2586%2592%25e2%2588%259e-frac-lnnn-sum-limits-k-1n-fraca-kk%23new-answer', 'question_page');
}
);

Post as a guest















Required, but never shown

























2 Answers
2






active

oldest

votes








2 Answers
2






active

oldest

votes









active

oldest

votes






active

oldest

votes









2












$begingroup$

Let's start with what we know. We are given that $lim_{n rightarrow infty}a_n=a$, but you will see that we don't actually need to use this. All that matters is that the sequence is bounded (convergence implies bounded). i.e.



$$exists ; M in Bbb R ; | ; |a_k|<M ; ; forall ; k in Bbb N$$



Also, the Euler constant should come in handy, and it is defined as $$gamma := lim_{n rightarrow infty}bigg(sum_{k=1}^nfrac 1k - ln(n)bigg)$$



and implies that



$$lim_{n rightarrow infty}bigg|sum_{k=1}^nfrac 1k - ln(n)-gammabigg|=0$$



Using L'Hopital's rule, we see that



$$lim_{n rightarrow infty}frac{big[ln(n)big]^2}{n}=0 qquad text{ and } qquad lim_{n rightarrow infty}frac{ln(n)}{n}=0 $$



Thus, we have:



begin{align}
& bigg|frac{ln(n)}{n}sum_{k=1}^nfrac {a_k}{k} bigg|\
leq & frac{ln(n)}{n}bigg|sum_{k=1}^nfrac {|a_k|}{k}bigg| \
leq & frac{ln(n)}{n}bigg|sum_{k=1}^nfrac {M}{k}bigg| \
= & Mfrac{ln(n)}{n}bigg(bigg|sum_{k=1}^nfrac {1}{k}-ln(n)-gamma bigg|+bigg|ln(n)+gammabigg|bigg) \
= & Mbigg(frac{ln(n)}{n}bigg)bigg|sum_{k=1}^nfrac {1}{k}-ln(n)-gamma bigg|+Mfrac{big[ln(n)big]^2}{n}+Mgamma frac{ln(n)}{n}\
rightarrow & M(0)|0| + M(0) + Mgamma (0) \
= & 0
end{align}



which implies that



$$lim_{n rightarrow infty}frac{ln(n)}{n}sum_{k=1}^nfrac {a_k}{k}=0$$






share|cite|improve this answer









$endgroup$













  • $begingroup$
    will there be a different method which does not involve so advance concepts
    $endgroup$
    – maveric
    Dec 23 '18 at 9:42










  • $begingroup$
    which of the above concepts do you consider advanced?
    $endgroup$
    – glowstonetrees
    Dec 23 '18 at 16:03
















2












$begingroup$

Let's start with what we know. We are given that $lim_{n rightarrow infty}a_n=a$, but you will see that we don't actually need to use this. All that matters is that the sequence is bounded (convergence implies bounded). i.e.



$$exists ; M in Bbb R ; | ; |a_k|<M ; ; forall ; k in Bbb N$$



Also, the Euler constant should come in handy, and it is defined as $$gamma := lim_{n rightarrow infty}bigg(sum_{k=1}^nfrac 1k - ln(n)bigg)$$



and implies that



$$lim_{n rightarrow infty}bigg|sum_{k=1}^nfrac 1k - ln(n)-gammabigg|=0$$



Using L'Hopital's rule, we see that



$$lim_{n rightarrow infty}frac{big[ln(n)big]^2}{n}=0 qquad text{ and } qquad lim_{n rightarrow infty}frac{ln(n)}{n}=0 $$



Thus, we have:



begin{align}
& bigg|frac{ln(n)}{n}sum_{k=1}^nfrac {a_k}{k} bigg|\
leq & frac{ln(n)}{n}bigg|sum_{k=1}^nfrac {|a_k|}{k}bigg| \
leq & frac{ln(n)}{n}bigg|sum_{k=1}^nfrac {M}{k}bigg| \
= & Mfrac{ln(n)}{n}bigg(bigg|sum_{k=1}^nfrac {1}{k}-ln(n)-gamma bigg|+bigg|ln(n)+gammabigg|bigg) \
= & Mbigg(frac{ln(n)}{n}bigg)bigg|sum_{k=1}^nfrac {1}{k}-ln(n)-gamma bigg|+Mfrac{big[ln(n)big]^2}{n}+Mgamma frac{ln(n)}{n}\
rightarrow & M(0)|0| + M(0) + Mgamma (0) \
= & 0
end{align}



which implies that



$$lim_{n rightarrow infty}frac{ln(n)}{n}sum_{k=1}^nfrac {a_k}{k}=0$$






share|cite|improve this answer









$endgroup$













  • $begingroup$
    will there be a different method which does not involve so advance concepts
    $endgroup$
    – maveric
    Dec 23 '18 at 9:42










  • $begingroup$
    which of the above concepts do you consider advanced?
    $endgroup$
    – glowstonetrees
    Dec 23 '18 at 16:03














2












2








2





$begingroup$

Let's start with what we know. We are given that $lim_{n rightarrow infty}a_n=a$, but you will see that we don't actually need to use this. All that matters is that the sequence is bounded (convergence implies bounded). i.e.



$$exists ; M in Bbb R ; | ; |a_k|<M ; ; forall ; k in Bbb N$$



Also, the Euler constant should come in handy, and it is defined as $$gamma := lim_{n rightarrow infty}bigg(sum_{k=1}^nfrac 1k - ln(n)bigg)$$



and implies that



$$lim_{n rightarrow infty}bigg|sum_{k=1}^nfrac 1k - ln(n)-gammabigg|=0$$



Using L'Hopital's rule, we see that



$$lim_{n rightarrow infty}frac{big[ln(n)big]^2}{n}=0 qquad text{ and } qquad lim_{n rightarrow infty}frac{ln(n)}{n}=0 $$



Thus, we have:



begin{align}
& bigg|frac{ln(n)}{n}sum_{k=1}^nfrac {a_k}{k} bigg|\
leq & frac{ln(n)}{n}bigg|sum_{k=1}^nfrac {|a_k|}{k}bigg| \
leq & frac{ln(n)}{n}bigg|sum_{k=1}^nfrac {M}{k}bigg| \
= & Mfrac{ln(n)}{n}bigg(bigg|sum_{k=1}^nfrac {1}{k}-ln(n)-gamma bigg|+bigg|ln(n)+gammabigg|bigg) \
= & Mbigg(frac{ln(n)}{n}bigg)bigg|sum_{k=1}^nfrac {1}{k}-ln(n)-gamma bigg|+Mfrac{big[ln(n)big]^2}{n}+Mgamma frac{ln(n)}{n}\
rightarrow & M(0)|0| + M(0) + Mgamma (0) \
= & 0
end{align}



which implies that



$$lim_{n rightarrow infty}frac{ln(n)}{n}sum_{k=1}^nfrac {a_k}{k}=0$$






share|cite|improve this answer









$endgroup$



Let's start with what we know. We are given that $lim_{n rightarrow infty}a_n=a$, but you will see that we don't actually need to use this. All that matters is that the sequence is bounded (convergence implies bounded). i.e.



$$exists ; M in Bbb R ; | ; |a_k|<M ; ; forall ; k in Bbb N$$



Also, the Euler constant should come in handy, and it is defined as $$gamma := lim_{n rightarrow infty}bigg(sum_{k=1}^nfrac 1k - ln(n)bigg)$$



and implies that



$$lim_{n rightarrow infty}bigg|sum_{k=1}^nfrac 1k - ln(n)-gammabigg|=0$$



Using L'Hopital's rule, we see that



$$lim_{n rightarrow infty}frac{big[ln(n)big]^2}{n}=0 qquad text{ and } qquad lim_{n rightarrow infty}frac{ln(n)}{n}=0 $$



Thus, we have:



begin{align}
& bigg|frac{ln(n)}{n}sum_{k=1}^nfrac {a_k}{k} bigg|\
leq & frac{ln(n)}{n}bigg|sum_{k=1}^nfrac {|a_k|}{k}bigg| \
leq & frac{ln(n)}{n}bigg|sum_{k=1}^nfrac {M}{k}bigg| \
= & Mfrac{ln(n)}{n}bigg(bigg|sum_{k=1}^nfrac {1}{k}-ln(n)-gamma bigg|+bigg|ln(n)+gammabigg|bigg) \
= & Mbigg(frac{ln(n)}{n}bigg)bigg|sum_{k=1}^nfrac {1}{k}-ln(n)-gamma bigg|+Mfrac{big[ln(n)big]^2}{n}+Mgamma frac{ln(n)}{n}\
rightarrow & M(0)|0| + M(0) + Mgamma (0) \
= & 0
end{align}



which implies that



$$lim_{n rightarrow infty}frac{ln(n)}{n}sum_{k=1}^nfrac {a_k}{k}=0$$







share|cite|improve this answer












share|cite|improve this answer



share|cite|improve this answer










answered Dec 23 '18 at 3:42









glowstonetreesglowstonetrees

2,368418




2,368418












  • $begingroup$
    will there be a different method which does not involve so advance concepts
    $endgroup$
    – maveric
    Dec 23 '18 at 9:42










  • $begingroup$
    which of the above concepts do you consider advanced?
    $endgroup$
    – glowstonetrees
    Dec 23 '18 at 16:03


















  • $begingroup$
    will there be a different method which does not involve so advance concepts
    $endgroup$
    – maveric
    Dec 23 '18 at 9:42










  • $begingroup$
    which of the above concepts do you consider advanced?
    $endgroup$
    – glowstonetrees
    Dec 23 '18 at 16:03
















$begingroup$
will there be a different method which does not involve so advance concepts
$endgroup$
– maveric
Dec 23 '18 at 9:42




$begingroup$
will there be a different method which does not involve so advance concepts
$endgroup$
– maveric
Dec 23 '18 at 9:42












$begingroup$
which of the above concepts do you consider advanced?
$endgroup$
– glowstonetrees
Dec 23 '18 at 16:03




$begingroup$
which of the above concepts do you consider advanced?
$endgroup$
– glowstonetrees
Dec 23 '18 at 16:03











1












$begingroup$

First consider the case where $a_k = a$ for all $k$. Then the sum becomes a sum of $1/k$, which can be approximated by comparing it to the integral of $1/x$ in some way that I forget but involves the Euler-Mascheroni constant.



Anyway I think you get something that converges to $0$, and then you can argue that if $a_k to a$ then it isn't different enough from the above case, so will also converge to $0$.






share|cite|improve this answer









$endgroup$













  • $begingroup$
    could not understand the last few lines.
    $endgroup$
    – maveric
    Dec 23 '18 at 3:04
















1












$begingroup$

First consider the case where $a_k = a$ for all $k$. Then the sum becomes a sum of $1/k$, which can be approximated by comparing it to the integral of $1/x$ in some way that I forget but involves the Euler-Mascheroni constant.



Anyway I think you get something that converges to $0$, and then you can argue that if $a_k to a$ then it isn't different enough from the above case, so will also converge to $0$.






share|cite|improve this answer









$endgroup$













  • $begingroup$
    could not understand the last few lines.
    $endgroup$
    – maveric
    Dec 23 '18 at 3:04














1












1








1





$begingroup$

First consider the case where $a_k = a$ for all $k$. Then the sum becomes a sum of $1/k$, which can be approximated by comparing it to the integral of $1/x$ in some way that I forget but involves the Euler-Mascheroni constant.



Anyway I think you get something that converges to $0$, and then you can argue that if $a_k to a$ then it isn't different enough from the above case, so will also converge to $0$.






share|cite|improve this answer









$endgroup$



First consider the case where $a_k = a$ for all $k$. Then the sum becomes a sum of $1/k$, which can be approximated by comparing it to the integral of $1/x$ in some way that I forget but involves the Euler-Mascheroni constant.



Anyway I think you get something that converges to $0$, and then you can argue that if $a_k to a$ then it isn't different enough from the above case, so will also converge to $0$.







share|cite|improve this answer












share|cite|improve this answer



share|cite|improve this answer










answered Dec 23 '18 at 2:40









Ben MillwoodBen Millwood

11.3k32049




11.3k32049












  • $begingroup$
    could not understand the last few lines.
    $endgroup$
    – maveric
    Dec 23 '18 at 3:04


















  • $begingroup$
    could not understand the last few lines.
    $endgroup$
    – maveric
    Dec 23 '18 at 3:04
















$begingroup$
could not understand the last few lines.
$endgroup$
– maveric
Dec 23 '18 at 3:04




$begingroup$
could not understand the last few lines.
$endgroup$
– maveric
Dec 23 '18 at 3:04


















draft saved

draft discarded




















































Thanks for contributing an answer to Mathematics Stack Exchange!


  • Please be sure to answer the question. Provide details and share your research!

But avoid



  • Asking for help, clarification, or responding to other answers.

  • Making statements based on opinion; back them up with references or personal experience.


Use MathJax to format equations. MathJax reference.


To learn more, see our tips on writing great answers.




draft saved


draft discarded














StackExchange.ready(
function () {
StackExchange.openid.initPostLogin('.new-post-login', 'https%3a%2f%2fmath.stackexchange.com%2fquestions%2f3050018%2ffinding-lim-limits-n%25e2%2586%2592%25e2%2588%259e-frac-lnnn-sum-limits-k-1n-fraca-kk%23new-answer', 'question_page');
}
);

Post as a guest















Required, but never shown





















































Required, but never shown














Required, but never shown












Required, but never shown







Required, but never shown

































Required, but never shown














Required, but never shown












Required, but never shown







Required, but never shown







Popular posts from this blog

Quarter-circle Tiles

build a pushdown automaton that recognizes the reverse language of a given pushdown automaton?

Mont Emei